An aircraft on a reconnaissance mission takes off from its home base and flies 550 miles at a bearing of s 46° e to a location in the sea. it then flies 483 miles from the sea at a bearing of s 55° w to another location. finally, the aircraft flies straight back from the second location to its base. what is the total distance it flies, rounded to the nearest mile? 2 vertical lines. top, point a (airport), point c below are marked on left line. point b marked on middle of right line. abc makes triangle. ab as c equals 5.5. bc as a equals 4.83. interior angle a 46 degrees. exterior angle b 55 degrees. a. 550 miles b. 1,033 miles c. 1,583 miles d. 1,692 miles e. 2,210 miles

Answers

Answer 1

The  total distance it flies, rounded to the nearest mile is: d. 1,692 miles .

Total distance

First distance= 550 miles (given)

Second distance=483 miles (given)

Now let find the third distance

Let x represent the third distance

Hence:

x/sin79 = 550/sin55

x= 659.1

Now let calculate the total distance

Total distance=550+483+659.1

Total distance= 1692.1

Total distance= 1692 miles (rounded)

Therefore the correct option is d.

Learn more about total distance here:https://brainly.com/question/4931057

#SPJ1


Related Questions

!!!!PLEASE HELP FAST !!!!

Solve each discrete exponential growth/decay problem. A country pledges to
reduce its CO2 emissions by 3.9% per year. If the emissions in 2022 are 5,160 Mt
(metric megatons), what are the maximum allowable emissions in the year 2030?
A) 5160 0.844≈ 1,329 Mt
B) 5160-0.961 = 3,754 Mt
C) 10320-0.961 = 7,507 Mt
D) 5160-0.99=4,761 Mt

Answers

Answer:

C) 10320-0.961 = 7,507 Mt

that is my answer please have a great day


What are the solutions of 2x² - 6x+5=0?

Answers

Answer:

no solutions

Step-by-step explanation:

we use the quadratic formula.

for an equation ax²+bx+c=0, the solutions for x are:

x = (-b±sqrt(b²-4ac))/2a

in our case, a=2, b=-6 and c=5, so:

x= (6±sqrt(36-40))/4

we see that we get a negative number inside the sqrt, so we have no real solutions

Answer:

No solution

Step-by-step explanation:

1) Use the quadratic formula

[tex]x=\frac{-b\pm\sqrt{b^2-4ac} }{2a}[/tex]

Once in standard form, identify a, b, and c from the original equation and plug them into the quadratic formula.

[tex]2x^2-6x+5=0[/tex]

[tex]a=2\\b=-6\\c=5[/tex]

[tex]x=\frac{-(-6)\pm\sqrt{(-6)^2-4*2*5} }{2*2}[/tex]

2) Simplify

Evaluate the exponent:

[tex]x=\frac{6\pm\sqrt{(-6)^2-4*2*5} }{2*2}[/tex]

[tex]x=\frac{6\pm\sqrt{36-4*2*5} }{2*2}[/tex]

Multiply the numbers:

[tex]x=\frac{6\pm\sqrt{36-4*2*5} }{2*2}[/tex]

[tex]x=\frac{6\pm\sqrt{36-40} }{2*2}[/tex]

Subtract the numbers:

[tex]x=\frac{6\pm\sqrt{36-40} }{2*2}[/tex]

[tex]x=\frac{6\pm\sqrt{-4} }{2*2}[/tex]

Multiply the numbers

[tex]x=\frac{6\pm\sqrt{-4} }{2*2}[/tex]

[tex]x=\frac{6\pm\sqrt{-4} }{4}[/tex]

3) No real solutions because the discriminant is negative

The square root of a negative number is not a real number

[tex]d=-4[/tex]

Result

No solution

p=f/a, where f=70 and a=20 f is increased by 10 a is increased to by 10

Answers

Answer:

P=3.5

P=2.667

Step-by-step explanation:

p=f/a

P=70/20

P=3.5

then f is increased by 10

f=70+10=80

and a is increased by 10

a=20+10

a=30

therefore; P=f/a

P=80/30

P=2.667

Find x intercept for n and find y intercept for m

Answers

The anwsers is 4,2 &3,4

This is direct proportion .

I need help in question 2 only one of the questions then I can do the rest tysm
Il give

Answers

Answer:

(a) 2.5

Step-by-step explanation:

(a) Since y is directly proportional to x we can write y and x in terms of the following equation:

y = cx where c is a constant
and c = y/x ...(1)
or,

x = y/c ...(2)

For (a)
y = 8 when x = 2 ==> c = y/x = 8/2 =4 from (1)

From (2) we get x = 10/4 = 2.5

The other questions can be solved in a similar fashion



Explain what the result of your proof tells you about angles a and b. Specifically, if you measured one angle, what would you know about the other? (2 points)

Answers

If the measure of one of the angles ∠a or ∠b is known, then the other angle is given by subtracting the known angle from 180°

What are supplementary angles?

Two angles whose sum is 180° are called supplementary angles. If a straight line is intersected by a line, then there are two angles form on each of the sides of the considered straight line.

The required two-column proof is presented as follows;

Statement                                                    Reason

∠1 and ∠a are linear pair angles                Given

∠1 + ∠a = 180°                                              Linear pair ∠s are supplementary

∠2 and ∠b are linear pair angles               Given

∠2 + ∠b = 180°                                             Linear pair ∠s are supplementary

∠a and ∠b are consecutive interior ∠s      Definition

x║y                                                               Given

∠1 and ∠b are corresponding angles        Definition

∠1 ≅ ∠b                                                        Corr. ∠s formed between║ lines

∠1 = ∠b                                                         Definition of congruency

∠a + ∠b = 180°                                              Substitution property

∠a and ∠b are supplementary                    Definition

We can conclude from the proof that given ∠1 and ∠b are located in corresponding locations relative to the common transversal of the parallel lines, they (∠1 and ∠b) are equal.

Therefore, the sum of ∠a and ∠b is 180°, given that the sum of ∠1 and ∠a is 180° by substitution property of equality. So, ∠a and ∠b are supplementary angles.

Therefore, If the measure of one of the angles ∠a or ∠b is known, then the other angle is given by subtracting the known angle from 180°

Learn more about supplementary angles here:

https://brainly.com/question/2882938

#SPJ1

Hey can I have some help with this. Just the answer is fine

Answers

I believe the answer would be:
4.5

Step-by-step explanation:

4.500 mm³ → cm³

1 cm³ = 1.000 mm³

= 4.500 ÷ 1.000

= 4,5 cm³ IS THE ANSWER

what is the answr to -5(b + 1) = 8

Answers

[tex] - 5(b + 1) = 8 \\ - 5b - 5 = 8 \\ - 5b = 8 + 5 \\ - 5b = 13 \\ b = \frac{ - 13}{5} [/tex]

PLEASE GIVE BRAINLIEST

\\\///\\\///\\\///\\\///\\\///\\\///\\///\\\///\\\///\\\///\\\///\\\//\\//\\//\\//\\\///\\//\\\///\\\///\\\///\

                                      [tex]\Rsh[/tex] Hey! [tex]\Lsh[/tex]

[tex]\Rsh[/tex]Let's solve this! We can do this! [tex]\Lsh[/tex]

>>> First, let's multiply -5 by b and 1. Then we obtain

[tex]\pmb{-5b-5=8}[/tex].

>>> Now we add 5 to both sides.

[tex]\pmb{-5b=13}[/tex]

>>> And then, divide both sides by -5.

[tex]\pmb{b=-\dfrac{13}{5}}[/tex]

>>> That is the value of b.

><><><><><<><><><><><><><><><><><><><><><><><><><><><><><><><><>

Hope the answer - and explanation - assisted you!

Happy studies! :)

What is the absolute value of -6 ?

Answers

Answer:

the answer is just 6

Step-by-step explanation:

Answer:

Step-by-step explanation:

An absolute value of a number is written like this | x |, where x could be any number that you want.

The absolute value is a math function that counts from your given number to 0.

Example: If you put the number 3, the function will count from 3 to 0, and the number will be 3.

| 3 | = 3

If you input the number 8, from 8 to 0 are 8 numbers. So, the output's function will be 8.

| 8 | = 8

If you input a negative number, for example, -6, the function will do the same thing. It will count from -6 to 0. From -6 to 0, are 6 numbers.

| -6 | = 6

Any number that you input in this function, will always output the same number, but positive

| -10 | = 10

| -46348 | = 46348

Hope I helped !

In the figure below, segment HK and segment IJ intersect at P, and segment HI is parallel to segment Jk. Which of the following angles must be congruent to angle PKJ?

Answers

Answer:

angle PHI

Step-by-step explanation:

Using the alternate interior angle theorem, we can figure out that PHI is congruent to PKJ

Solve this system of equations by
using the elimination method.
2x - 3y = 25
5x + 3y = 10
The ordered pair of solutions
is written in the format (x, y).
([?], [])
Enter

Answers

Answer:

(5,-5)

x=5

y=-5

Step-by-step explanation:

How to solve using elimination method.

First, add the two equations.

   2x - 3y = 25

+  5x + 3y = 10

7x=35

The y values got eliminated.

Next, divide 7 from both sides.

7x/7=35/7

x=5

Next, plug-in x=5 into one of the equations.

2(5) - 3y = 25

10-3y=25

Subtract 10 from both sides.

-3y=15

Divide -3 from both sides.

y=-5

Remember a positive divided by a negative is a negative.

Hope this helps!

If not, I am sorry.

what does parallelepiped focus on

Answers

Answer:

A parallelepiped has three sets of four parallel edges; the edges within each set are of equal length. Parallelepipeds result from linear transformations of a cube (for the non-degenerate cases: the bijective linear transformations).

Step-by-step explanation:

A parallelepiped focuses on three-dimensional figures formed by six parallelograms.

How to explain the parallelepiped?

A parallelepiped shape has the following features

They are three-dimensional figuresThey are formed by 6 parallelogramsOpposite parallelograms are congruent

Think of a cube or a cuboid.

Notice that they contain squares and rectangles

A parallelepiped is to a parallelogram as a square is to a cube and a rectangle to a cuboid

Read more about parallelepiped at:

https://brainly.com/question/3050890

#SPJ11

The international air transport association surveys business travelers to develop quality ratings for transatlantic gateway airports. the maximum possible rating is 10. suppose a simple random sample of business travelers is selected and each traveler is asked to provide a rating for the miami international airport. the ratings obtained from the sample of business travelers follow. 8 8 4 0 5 5 5 4 4 4 4 3 10 6 10 10 0 8 5 4 3 2 4 7 8 9 10 8 4 5 5 4 4 3 8 9 9 5 3 9 8 8 5 10 4 10 5 5 3 3 develop a confidence interval estimate of the population mean rating for miami. round your answers to two decimal places. ( , )

Answers

Using the t-distribution, the 95% confidence interval of the population mean rating for Miami is (4.97, 6.51).

What is a t-distribution confidence interval?

The confidence interval is:

[tex]\overline{x} \pm t\frac{s}{\sqrt{n}}[/tex]

In which:

[tex]\overline{x}[/tex] is the sample mean.t is the critical value.n is the sample size.s is the standard deviation for the sample.

The parameters for this problem are given as follows:

[tex]\overline{x} = 5.74, s = 2.7, n = 50[/tex]

The critical value, using a t-distribution calculator, for a two-tailed 95% confidence interval, with 50 - 1 = 49 df, is t = 2.0096.

Hence the bounds of the interval are given as follows:

[tex]\overline{x} - t\frac{s}{\sqrt{n}} = 5.74 - 2.0096\frac{2.7}{\sqrt{50}} = 4.97[/tex]

[tex]\overline{x} + t\frac{s}{\sqrt{n}} = 5.74 + 2.0096\frac{2.7}{\sqrt{50}} = 6.51[/tex]

More can be learned about the t-distribution at https://brainly.com/question/16162795

#SPJ1

Select the correct answer.
Which graph best models the inequality?
y ≤ -2/5x + 2

Answers

Answer:

it would look like this

Step-by-step explanation:

Please help meeeeeeeeee???

Answers

The answer for the exponential expressions 1,2,3 and 4 will be [tex]4.5,\frac{1}{2},2,1[/tex] respectively.

What exactly is an exponent?

Exponential notation is a type of mathematical shorthand that allows us to write complex formulas in a more concise manner.

The basis of an exponent is a number or letter. It means that the base will be raised to a given level of power. The base is X, and the power is n.

The properties of the exponent used;

[tex]\rm m^a \times m^b = m^{a+b} \\\\ m^a \times n^a = {mn}^a \\\\ m^a \times m^b = m^{a-b} \\\\ m^0 = 1[/tex]

Expression 1;

[tex]\frac{(2.3^{-2})^2(5.3^2.2)^2}{(3)^-2(5.2)^2} \\\\ \frac{2^3.3^{-6}.5^2.3^6}{3^{-2}.5^2.2^2} \\\\ 4.5[/tex]

Expression 2;

[tex](3^3)(4^0)(3.2)^{-3}(2)^2 \\\\ 3^3.1.3^{-3}.2^{-3}.2^2 \\\\ \frac{1}{2}[/tex]

Expression 3;

[tex]\frac{(3^7.4^7)(2.5)^{-3}(5)^2}{12^7.5^{-1}.2^{-4}} \\\\ \frac{3^7.4^7.25^{-3}.5^{-3}.5^2}{12^7.5^{-1}2^{-4}} \\\\ 2[/tex]

Expression 4;

[tex]\frac{(2.3)^{-1}.2^0}{(2.3)^{-1}}\\\\ \frac{2^{-1}.3^{-1}.1}{2.3}\\\\ 1[/tex]

Hence, the answer for the exponential expressions 1,2,3, and 4 will be [tex]4.5,\frac{1}{2},2,1[/tex] respectively.

To learn more about exponents, see

https://brainly.com/question/5497425

#SPJ1

The World Health Organization lists the following overall literacy rates per 100 people for countries. Which answer choice includes all countries with literacy rates above 90%?

Answers

Answer:

The answer is D

Cuba, Poland, U.S., Argentina, Turkey, and Spain

Step-by-step explanation:

Basically, what it was asking, is what countries are over 90%? So, all you have to do - to put it simply - is just look at the countries that's numbers are greater than 90%

what is mean by Expanded form power of 10 please answer it quickly urgent!

Answers

The expanded form of a power of ten is like modeling the expanded for mhm of a normal number but in terms of powers of ten. Here, I will demonstrate by showing you an example. Here is the expanded form of 1,000:

1,000.

Why? Well, the expanded form of 1,000 is itself because it is in a digits place notable for being able to do expanded form in. There are the ones place, tens place, hundreds place, and thousands place. Because ten to the third power is 1,000, the number itself is representative of its own expanded form. This varies for numbers that are different though. Another example would be as follows; the number 1,783. Here is the expanded form of that number:

1,000 + 700 + 80 + 3

Remember the digits places I mentioned above (ones, tens, hundreds, and thousands)? Those come in handy for these kinds of numbers. 1 is in the thousands place so the number is 1,000. 7 is in the hundreds place so the number is 700. 80 is in the tens place so the number is 80. Lastly, 3 is in the ones place so the number is 3. An easy way to remember expanded form is to just take the number by what digits place it is in, and multiply it by the corresponding digit value that it is in. For example, to get 80, we can do 8 x 10 because we are multiplying the 8 that is in the tens place by the digit place it is in, which happens to be the tens place. If you need to better understand, let me know and I will gladly assist you!

Explain how to find the average rage of change for a function over a given interval.

Answers

The average rate of change for a function over an interval is the slope

How to explain the steps?

Take for instance, we have the function to be

Function f(x)

And the interval is (a,b)

The average rate of change for the function over the interval is

Rate = (f(b) - f(a))/(b - a)

The above represents the slope

Take for instance, we have:

f(x) = x^2 [2, 4]

We have:

f(2) = 2^2 = 4

f(4) = 4^2 = 16

So, we have:

Rate = (16 - 4)/(4 - 2)

Rate = 6

Read more about average rate of change at:

https://brainly.com/question/8728504

#SPJ1

Which table can be created using the equation below?
-2 + 4x = y
X
y
-5-22
0
3
X
y
-22 -5
-2 0
10
3
X
-5
0
-2
10
03
y
-18
-2
10

Answers

Answer:

h

Step-by-step explanation:

h

Eleanor randomly chooses a number from 1 to 10. what is the probability she chooses a number greater than 5?

Answers

The probability she chooses a number greater than 5 will be 1/2 or 0.5.

What is probability?

Its simple notion is that something will most likely occur. The favorable event's proportion to the overall number of occurrences.

Probability = (Favorable event) / (Total event)

Eleanor randomly decides a number from 1 to 10.

Then the probability she chooses a number greater than 5 will be

Total event = 10

Favorable event = 5 {6, 7, 8, 9, 10}

Then we have

P = 5/10

P = 1/2

P = 0.5

More about the probability link is given below.

https://brainly.com/question/795909

#SPJ1

Which equation is the inverse of y = 9x² - 4?​

Answers

Answer:

y = ± [ √(x+4) ] / 3

Step-by-step explanation:

1) To find the inverse of the function y = 9x2 – 4, exchange the variables x and y, and then solve for the new y.

2) Exchange of the variables: y = 9x^2 – 4 → x = 9y^2 – 4

3) Solve for y:

=> 9y^2 = x + 4

=> y^2=> y = ± [ √(x + 4) ] / 9

= > y = ± [ √(x + 4) ] / 3

Help with Pre Calculus

Answers

the answer is the first one .. y=3x-4

Use the graph of f(x) to find the indicated function values.

Answers

The indicated function values are f(0) = 2 and x = -1

How to determine the function values?

The attached figure represents the missing piece in the question

From the attached figure, the function values to calculate are:

f(x) when x = 0 and x when f(x) = 0

From the figure, the graph crosses the y-axis at y = 2 and the x-axis at x = -1

This means that:

f(0) = 2 and f(-1) = 0

Hence, the indicated function values are f(0) = 2 and x = -1

Read more about functions at

https://brainly.com/question/12518981

#SPJ1

what is an equivalent expression ​

Answers

Answer:

1st option

Step-by-step explanation:

using the rules of logarithms

• log x + log y = log xy

• n logx ⇔[tex]x^{n}[/tex]

given

2[tex]log_{2}[/tex] 3 + [tex]log_{2}[/tex] 3

= [tex]log_{2}[/tex] 3² + [tex]log_{2}[/tex] 3

= [tex]log_{2}[/tex] 9 + [tex]log_{2}[/tex] 3

= [tex]log_{2}[/tex] (9 × 3)

= [tex]log_{2}[/tex] 27

whenx=2y=50 and whenx=4y=100 Which direct variation equation can be used to model this function?

Answers

The equation which is used to model this function is y = 25x.

What is equation?

An equation is a mathematical expression that contains an equals symbol. Equations often contain algebra.

Given:

x=2 , y=50

x=4, y=100

As, corresponding to x =2 , y =50

For x=1 , y=25

Hence, the equation which is used to model this function is y = 25x.

Learn more about equation here:

https://brainly.com/question/10413253

#SPJ1

Please help me with this thanks!!

Answers

Answer: 47°

Step-by-Step Explanation:

In a Triangle the Sum of all the Angles equal to 180° (Angle Sum Property)

Therefore,
x + 78 + 55 = 180
x + 133 = 180
x = 180 - 133
=> x = 47

Hence, ∠x = 47°

Answer:

x° = 47°

Step-by-step explanation:

The sum of the measurements of three interior Angles in a triangle is 180°.This is known as angle sum property.Same case is here.

78°,55° and x° are interior angles here.

So,

[tex] 78{}^{ \circ} + 55 {}^{ \circ} + x {}^{ \circ} = 180 {}^{ \circ} [/tex]

Solving for x°,

[tex]133 {}^{ \circ} + x {}^{ \circ} = 180 {}^{ \circ} [/tex][tex]x {}^{ \circ} = 180 {}^{ \circ} - 133 {}^{ \circ} [/tex][tex]x {}^{ \circ} = \boxed{47 {}^{ \circ} }[/tex]

Hence,the value of x° is 47°.

Carter paid $201.75 for a camera and a memory card. if the 32 gb memory
card costs $12.35, and a discount of 20% is given to the camera, find the
regular price of the camera. estimate it to the nearest whole number.

Answers

The regular price of the camera is $227.

Given that, the total amount paid=$201.75 and the cost of the memory card=$12.35.

Cost of camera=201.75-12.35=$189.4

What is a discount?

A discount is a deduction from the usual price of something. To discount means to deduct an amount from the price.

Discount is given to the camera=20%.

The regular price of the camera=189.4+20% of 189.4

=189.4+0.2×189.4=189.4+37.88=$227.28

The estimation to the nearest the whole number is $227.

Therefore, the regular price of the camera is $227.

To learn more about discount visit:

https://brainly.com/question/3541148.

#SPJ1

answer

yes do it your self

This is the last question PLSSSSS HELP

Answers

Answer:

D

Step-by-step explanation:

The volume of a cylinder is pi * r^2 * h

A) V = 3.14 * (0.7/2)^2 * 6.2 = 2.4 m^2

B) V = 3.14 * (0.9/2)^2 * 3.4 = 2.2 m^2

C) V = 3.14 * (1.2/2)^2 * 6.8 = 7.7 m^2

D) V = 3.14 * (1.4/2)^2 * 6.2 = 9.5 m^2

Which is a true statement about an exterior angle of a triangle

Answers

Answer:

would love to help but what  are the statements?

3.5 decimals to mixed numbers

Answers

1 2/3
It’s one and two thirds written out
Other Questions
Question 3What part of a water molecule is the negatively charged side? Which expression is equivalent to 4(x+2)? The percentage of automobile consumers who are under 50 years of age decreased approximately linearly from 58.8% in 1980 to 50.5% in 1995.(A) Predict when the percentage will be 47%(B) Predict the percentage in 2005. The table below shows the average SAT math scores from 1993-2002.YearSAT math scores19935031994504199550619965081997511199851219995112000514Using the data from the table determine if there is a linear trend between the year and the average SAT math scores and determine if there is an exact linear fit of the data. Describe the linear trend if there is one.a.Positive linear trend, an exact linear fit.b.Positive linear trend, not an exact linear fit.c.Negative linear trend, not an exact linear fit.d.Negative linear trend, an exact linear fit.Please select the best answer from the choices providedABCDMark this and return Question 8 of 10Calculate the electric potential energy in a capacitor that stores 1.0 x 10-10 Cof charge at 100.0 V. Use PE =2 What happened to the indigenous peoples who fought in the french and indian war? Explain the propagation of electrical impulses along a neuron including the role of myelin PLEASE HELP ASAP!!!!The polar equation r=7sin(2) graphs as a rose.What is the length of the petals of this rose?Enter your answer in the box. Which events happen first in Edgar Allan Poe's "The Black Cat biology lab 14 experiment 1 punnett square crosses post lab questions A set of photographs has been taken for a wedding. All the guests are to be sent digitallystored copies through the ordinary postal service. There are fifty photographs and eachphotograph is between 1.8 and 2.5 megabytes in size.Work out the maximum storage space required for a set of photographs. State, with areason, a suitable medium to use for the copies to be sent to the guests. How might the State Department policies change from president to president, and would this affect our standing in the world? 2. Say for some generic good that the equilibrium price fell and the equilibrium quantity remained constant. What would you say was the most likely cause? An empty container has a mass of 62.000 grams. You fill it with 24.6 mL of a liquid. The final mass of the container is 81.34 grams. What is the density of the liquid to the correct number of significant figures? Which represents the reflection of f(x) = StartRoot x EndRoot over the y-axis?A 2-column table has 4 rows. The first column is labeled x with entries negative 1, 0, 1, 4. The second column is labeled f (x) with entries undefined, 0, 1, 2.A 2-column table has 4 rows. The first column is labeled x with entries negative 1, 0, 1, 4. The second column is labeled f (x) with entries undefined, 0, negative 1, negative 2.On a coordinate plane, an absolute value graph starts at (0, 0) and goes to the left through (negative 4, 2).On a coordinate plane, an absolute value graph starts at (0, 0) and goes up and to the left through (negative 2, 4). What are the exact solutions of x2 3x 5 = 0? (1 point) a x = the quantity of negative 3 plus or minus the square root of 29 all over 2 b x = the quantity of 3 plus or minus the square root of 29 all over 2 c x = the quantity of 3 plus or minus the square root of 11 all over 2 d x = the quantity of negative 3 plus or minus the square root of 11 all over 2 You are trying to establish a Wi-Fi Direct connection between a Windows desktop system and a Windows tablet device. You have installed an 802.11n wireless adapter in the desktop system. However, you are unable to see the tablet device. Which action must you take to see the tablet device from the desktop 0.467 mol NaCl are needed tocreate a 0.875 m solution using 534g of water. How many grams ofsodium chloride are required tomake the solution? Which details from the excerpt best indicate that the tomb is a frightening place? select 3 options. For a given interest rate, simple interest varies jointly as principal and time. If $2000 left in an account for 4 yr earned interest of $280, how much interest would be earned in 6 yr?